Derivación de la ecuación geodésica utilizando un multiplicador de Lagrange para corregir la parametrización afín

La ecuación geodésica se puede derivar usando la acción

(1) S 0   =   d τ X ˙ m X ˙ m
(Estoy usando la convención (-+++) y X ˙ = d X d τ ). Para simplificar los cálculos, se elige una parametrización explícita, a saber, la longitud del arco τ es decir
(2) X ˙ m X ˙ m = 1.
Desde mi punto de vista esto significa que minimizo la acción con la restricción:
(3) X ˙ m X ˙ m + 1 = 0.
Entonces, la ecuación resultante debería ser la misma si uso la siguiente acción en su lugar
(4) S = d τ X ˙ m X ˙ m + λ ( X ˙ m X ˙ m + 1 )
dónde λ es un multiplicador de Lagrange.

Encontremos el eom en el espacio de Minkowski:

(5) 0 = pag ˙ m = d d τ ( X ˙ m X ˙ m X ˙ m + 2 λ X ˙ m )
(6) X ˙ m X ˙ m + 1 = 0.

La raíz cuadrada en la primera ecuación es igual a 1. Entonces

(7) pag m = ( 2 λ 1 ) X ˙ m .
De la segunda ecuación encuentro
X ¨ m X ˙ m = 0.
Usando esto
(8) d d τ X ˙ m pag m = X ¨ m pag m + X ˙ m pag ˙ m = 0.
Entonces
(9) C o norte s t . = X ˙ m pag m = 1 2 λ λ ˙ = 0
Juntando todo se obtiene:
(10) pag ˙ m = ( 2 λ 1 ) X ¨ m = 0.

En el caso λ 1 2 esto simplemente da el viejo eom X ¨ = 0 . Sin embargo en el caso λ = 1 2 no hay restricción para X ¨ .

no entiendo donde esta el caso λ = 1 2 viene de. ¿Cómo lo trato? ¿Puedo simplemente descuidarlo? ¿O he olvidado algo?

Respuestas (1)

  1. Antes que nada, debemos enfatizar que lo que OP llama τ no es tiempo adecuado fuera de la cáscara, pero solo alguna parametrización de línea mundial (WL). Sin embargo, la restricción

    (A) X ˙ m X ˙ m     1
    implicará que el parámetro WL τ es el tiempo adecuado en la concha.

  2. Como la EOM depende de la primera derivada d λ d τ del multiplicador de Lagrange, debemos especificar una sola condición, por ejemplo, una condición inercial (IC) para λ . Si elegimos el IC diferente de 1 / 2 , evitamos el problema cuando λ es 1 / 2 .

  3. la naturaleza de la λ = 1 / 2 la patología es una degeneración de la cuestión de fuerza de restricción/rango faltante. Para ver esto más claramente, tenga en cuenta que podemos obtener una acción equivalente

    (B) S ~   =   τ i τ F d τ ( 1 + λ ( X ˙ m X ˙ m + 1 ) )
    insertando la restricción (A) en el primer término de la acción de OP (4). Si repetimos el cálculo de OP para la acción equivalente S ~ veremos que el problema se ha desplazado a λ = 0 . Claramente, el caso λ = 0 corresponde a un caso degenerado donde el principio de acción estacionario (B) está mal definido.

--

Si el parámetro WL τ es el tiempo adecuado fuera de la cáscara también, significaría que la acción de OP (4) es solo S = τ F τ i , que está fijado por las condiciones de contorno (BC). En otras palabras, la acción no dependería de la WL, es decir, el problema variacional estaría mal definido.

2. ok, ese es un buen punto. Pero supongo que este problema también ocurrirá en otros Lagrangianos. ¿Hay alguna manera de encontrar un valor no crítico de λ ¿en general? 3. ¿Hay alguna forma de arreglar esta degeneración? ¿Te gusta agregar otra restricción?
2. No, dependerá del problema variacional. 3. Sí. Ver punto 2.
En realidad no veo de dónde viene esta degeneración. Esto no sucede cuando se utilizan multiplicadores de Lagrange para minimizar una función. F : R norte R con restricciones. ¿Por qué sucede aquí? En lo que me impide elegir λ = 1 2 ?
Un fenómeno similar aparece si tratamos de encontrar puntos estacionarios para una función de la forma
F ( gramo ( X ) ) + λ gramo ( X ) , X R norte .
¿Sería posible resolver esto exigiendo que las ecuaciones de Euler Lagrange se cumplan para cada λ ?
Eso sería en general una demanda demasiado fuerte.
Al calcular el propagador de fotones en QED, se agrega la restricción de fijación 1 2 ξ m A m al lagrangiano. Este es exactamente el mismo ansatz, ¿no es así? El resultado es algo como η m v ( 1 ξ ) pag m pag v pag 2 . En mi conferencia se nos dijo que podemos elegir cualquier valor para ξ queremos (lo llamaron libertad de medida). De hecho, los elementos de la matriz de dispersión son independientes de ξ (esto se puede mostrar usando la identidad del barrio). ¿Por qué la fijación del indicador funciona para la electrodinámica pero no para la ecuación geodésica?